Transforming Triangle 2

Geometry Level 4

If Brilli the Ant is inside an equilateral triangle and 5 5 , 35 3 35\sqrt{3} , and 10 37 10\sqrt{37} units away from its vertices, how many units long is one side of the equilateral triangle?

Inspiration


The answer is 65.

This section requires Javascript.
You are seeing this because something didn't load right. We suggest you, (a) try refreshing the page, (b) enabling javascript if it is disabled on your browser and, finally, (c) loading the non-javascript version of this page . We're sorry about the hassle.

9 solutions

David Vreken
Jan 17, 2019

From the inspiration problem , we discovered that if P P is a point inside an equilateral triangle Q R S \triangle QRS , then equilateral triangles can be taken out of Q R S \triangle QRS and the remaining triangles can be rearranged to make a triangle with double the sides of P Q PQ , P R PR , and P S PS as shown below, which means that Q P R \angle QPR , Q P S \angle QPS , and R P S \angle RPS are exactly 60 ° 60° more than corresponding angles in the non-equilateral triangle.

In this case, double P Q PQ , P R PR , and P S PS are 10 10 , 70 3 70\sqrt{3} , are 20 37 20\sqrt{37} , and since 1 0 2 + ( 70 3 ) 2 = ( 20 37 ) 2 10^2 + (70\sqrt{3})^2 = (20\sqrt{37})^2 , these form a right triangle where the angle between 10 10 and 70 3 70\sqrt{3} is 90 ° 90° . Therefore, the angle between 5 5 and 35 3 35\sqrt{3} near Brilli the Ant is 90 ° + 60 ° = 150 ° 90° + 60° = 150° , and using the law of cosines, the third side (and also the side of the equilateral triangle) solves to 5 2 + ( 35 3 ) 2 2 5 35 3 cos 150 ° = 65 \sqrt{5^2 + (35\sqrt{3})^2 - 2 \cdot 5 \cdot 35\sqrt{3} \cos 150°} = \boxed{65} .

If only I had noticed the right triangle I could have avoided using the inverse law of cosines.

I found the smallest angle of the triangle using inverse law of cosines. Then I expanded it by 60 (as you solved the inspiration problem) and used the law of cosines on this new angle. Same result but more work.

Jeremy Galvagni - 2 years, 4 months ago

The more "natural" way (and commonly used in olympiad problems) to see this is to consider an anticlockwise rotation of 6 0 60^ \circ about Q Q that brings P P to P P ' . Then P P Q P P' Q is an isosceles triangle with vertex angle 6 0 60 ^ \circ , hence is equilateral. So P P S P P' S is a triangle with side lengths 5 , 35 3 , 10 37 5, 35 \sqrt{3} , 10 \sqrt{37} , hence is a right triangle.

This allows us to generalize to the general case (as does your approach), where we might not have a right triangle. We can calculate cos Q P S = cos ( S P P + 6 0 ) \cos \angle QPS = \cos ( \angle SPP' + 60 ^ \circ ) , and have cos S P P \cos \angle SPP' from the side lengths of the triangle.

Calvin Lin Staff - 2 years, 4 months ago

Log in to reply

That is very clever!

David Vreken - 2 years, 4 months ago

Is there an unstated assumption in the problem?

I worked in the complex plane and set one vertex at (5,0) and then set up equations which I (well... mathematica actually) could easily solve numerically for the other two vertices. I thus found TWO solutions for the side length (depending on the initial estimates). One is indeed the lovely solution 65, and the other one was 56.3471. I presume in this case Brilli is OUTSIDE the triangle!

Peter Macgregor - 2 years, 4 months ago

Log in to reply

You're right! I added the restriction that Brilli has to be inside the equilateral triangle.

David Vreken - 2 years, 4 months ago

Good catch. With the rotation argument (and placing the point outside the triangle), we get that the other length is

5 2 + ( 35 3 ) 2 + 2 5 35 3 cos 150 ° = 56.35 \sqrt{5^2 + (35\sqrt{3})^2 + 2 \cdot 5 \cdot 35\sqrt{3} \cos 150°} = \boxed{56.35} .

Hint: I will leave you to figure out why it's a "+". When you work through the proof, it should become obvious.

Calvin Lin Staff - 2 years, 4 months ago

Log in to reply

For the point outside the triangle, the rotation method shows us that we subtract 60 ° 60° instead of adding it, so cos Q P S = cos ( S P P 60 ° ) = cos ( 90 ° 60 ° ) = cos 30 ° = cos 150 ° \cos \angle QPS = \cos(\angle SPP' - 60°) = \cos(90° - 60°) = \cos 30° = -\cos 150° , which makes a "+" when inserted in the law of cosines.

David Vreken - 2 years, 4 months ago

Slight typo in the last equation. It should be

5 2 + ( 35 3 ) 2 2 5 35 3 cos 150 ° = 65 \sqrt{5^2 + (35\sqrt{3})^2 - 2 \cdot 5 \cdot 35\sqrt{3} \cos 150°} = \boxed{65} .

Calvin Lin Staff - 2 years, 4 months ago

Log in to reply

Thanks, I fixed it!

David Vreken - 2 years, 4 months ago
Chew-Seong Cheong
Jan 17, 2019

Let the side length of the equilateral triangle be a a and the coordinates of the top, left and right vertices be ( 0 , 3 2 a ) \left(0, \frac {\sqrt 3}2a\right) , ( 1 2 a , 0 ) \left(-\frac 12a, 0 \right) , and ( 1 2 a , 0 ) \left(\frac 12a, 0 \right) respectively, and the coordinates where Brilli is ( x , y ) (x,y) . Then by Pythagorean theorem :

{ ( x 0 ) 2 + ( y 3 2 a ) 2 = 5 2 x 2 + y 2 3 a y + 0.75 a 2 = 25 . . . ( 1 ) ( x + 1 2 a ) 2 + ( y 0 ) 2 = ( 10 37 ) 2 x 2 + a x + 0.25 a 2 + y 2 = 3700 . . . ( 2 ) ( x 1 2 a ) 2 + ( y 0 ) 2 = ( 35 3 ) 2 x 2 a x + 0.25 a 2 + y 2 = 3675 . . . ( 3 ) \begin{cases} (x-0)^2 + \left(y-\frac {\sqrt 3}2a\right)^2 = 5^2 & \implies x^2 + y^2 - \sqrt 3ay + 0.75a^2 = 25 & ...(1) \\ \left(x+\frac 12a\right)^2 + (y-0)^2 = (10\sqrt {37})^2 & \implies x^2 + ax + 0.25a^2 + y^2 = 3700 & ...(2) \\ \left(x-\frac 12a\right)^2 + (y-0)^2 = (35\sqrt 3)^2 & \implies x^2 - ax + 0.25a^2 + y^2 = 3675 & ...(3) \end{cases}

From ( 2 ) ( 3 ) : 2 a x = 25 a x = 12.5 . . . ( 4 ) (2)-(3): \implies 2ax = 25 \implies ax = 12.5 \quad ...(4)

From ( 2 ) + ( 3 ) : 2 x 2 + 0.5 a 2 + 2 y 2 = 7375 x 2 + y 2 = 3687.5 0.25 a 2 . . . ( 5 ) (2)+(3): \implies 2x^2 + 0.5a^2 + 2y^2 = 7375 \implies x^2 + y^2 = 3687.5 - 0.25a^2 \quad ...(5)

From ( 2 ) ( 1 ) : a x + 3 a y 0.5 a 2 = 3675 3 a y = 3662.5 + 0.5 a 2 . . . ( 6 ) (2)-(1): \implies ax + \sqrt 3ay - 0.5a^2 = 3675 \implies \sqrt 3a y = 3662.5 + 0.5a^2 \quad ...(6)

Therefore, we have:

( 3 a x ) 2 + ( 3 a y ) 2 = ( 12.5 3 ) 2 + ( 3662.5 + 0.5 a 2 ) 2 3 a 2 ( x 2 + y 2 ) = 468.75 + 13413906.25 + 3662.5 a 2 + 0.25 a 4 ( 5 ) : x 2 + y 2 = 3687.5 0.25 a 2 3 a 2 ( 3687.5 0.25 a 2 ) = 13414375 + 3662.5 a 2 + 0.25 a 4 \begin{aligned} {\color{#3D99F6}(\sqrt 3ax)^2} + \color{#D61F06}(\sqrt 3ay)^2 & = {\color{#3D99F6}(12.5\sqrt 3)^2} + \color{#D61F06} (3662.5 + 0.5a^2)^2 \\ 3a^2\color{#3D99F6}(x^2+y^2) & = 468.75 + 13413906.25 + 3662.5a^2 + 0.25a^4 & \small \color{#3D99F6} (5): \ x^2 + y^2 = 3687.5 - 0.25a^2 \\ 3a^2\color{#3D99F6}(3687.5 - 0.25a^2) & = 13414375 + 3662.5a^2 + 0.25a^4 \end{aligned}

a 4 7400 a 2 + 13414375 = 0 ( a 2 4225 ) ( a 2 3175 ) = 0 \begin{aligned} \implies a^4 - 7400a^2 + 13414375 & = 0 \\ (a^2 - 4225)(a^2-3175) & = 0 \end{aligned}

a = { 65 5 127 \implies a = \begin{cases} 65 \\ 5\sqrt{127} \end{cases}

Since 5 127 < 35 3 < 10 37 5\sqrt{127}<35\sqrt 3 < 10\sqrt{37} , the answer is a = 65 a=\boxed{65} .

Great solution!

David Vreken - 2 years, 4 months ago

Niranjan Khanderia - 2 years, 4 months ago

I solved it as an algebra problem in 3 unknowns: the x x and y y coordinates of Brilli and s s , the length of the side of the equilateral triangle with vertices at { 0 , 0 } \{0,0\} , s { 1 2 , 3 2 } s\{\frac12,\frac{\sqrt{3}}{2}\} and s { 1 , 0 } s\{1,0\} . Unfortunately, there are two solutions. On the basis that the problem implicitly specified an integer side length and that Brilli was inside the triangle, I selected the 65 side length as the submitted solution. The two solutions are: { { s 65 , x 425 13 , y 385 3 13 } , { s 5 127 , x 320 127 , y 350 3 127 } } \left\{\left\{s\to 65,x\to \frac{425}{13},y\to \frac{385 \sqrt{3}}{13}\right\},\left\{s\to 5 \sqrt{127},x\to \frac{320}{\sqrt{127}},y\to 350 \sqrt{\frac{3}{127}}\right\}\right\} .

Yes, s = 5 127 s = 5 \sqrt{127} would be a solution if Brilli was outside the triangle.

David Vreken - 2 years, 4 months ago
Ajit Athle
Jan 23, 2019

Great solution!

David Vreken - 2 years, 4 months ago

Assume x as the side.
Write Hero's formula for the areas of the three tangles.
Sum of this three formulas = 3 4 x 2 \dfrac{\sqrt{3}}4*x^2 .
Solving x=65 is only greater than 10 35 , a n d 35 3 10*\sqrt{35},~ and~35*\sqrt3 .


This is a great, unique solution! Thanks for sharing!

David Vreken - 2 years, 4 months ago

Log in to reply

Thank you.

Niranjan Khanderia - 2 years, 4 months ago
Aravind Vishnu
Jan 24, 2019

Let x x be the length of one side of the equilateral triangle.

Let θ \theta be the angle between sides of length 5 5 and 35 3 35\sqrt{3} .

By rule of cosines, we have,

x 2 = 5 2 + ( 35 3 ) 2 2 × 35 3 × 5 × cos θ x^2=5^2+(35\sqrt{3})^2-2\times 35\sqrt{3}\times 5\times \cos{\theta}

We have that the three given lengths together form the side of a right-angled triangle, Since

5 2 + ( 35 3 ) 2 = ( 10 37 ) 2 5^2+(35\sqrt{3})^2=(10\sqrt{37})^2 .

Therefore in the new triangle formed from the Inspiration Problem we have that the angle between sides of length 5 5 and 35 3 = 9 0 35\sqrt{3}=90^{\circ} . This implies (from answer to Inspiration Problem ), that θ = 15 0 \theta=150^{\circ}

Hence the first equation yields x 2 = 5 2 + ( 35 3 ) 2 2 × 35 3 × 5 × cos 15 0 = 25 + 3675 2 × 35 3 × 5 × ( cos ( 3 0 ) ) = 25 + 3675 + 525 = 4225 x^2=5^2+(35\sqrt{3})^2-2\times 35\sqrt{3}\times 5\times \cos{150^{\circ}}=25+3675-2\times 35\sqrt{3}\times 5\times(-\cos(30^{\circ}))=25+3675+525=4225

x = 65 \implies x=65

Zhang Xiaokang
Jan 30, 2019

Consider the original triangle ABC, rotate it counter clockwise 60 degrees to form the corresponding points A'B'C' Now γ = β upon rotation, so α + γ = α + β = 60° From which we can deduce PP'B is an equilateral triangle so PP' = BP = 5 Using the Cosine Rule, ∠BPP' turns out to be 90° so ∠BPC = 150° and the length of a side of the equilateral triangle BC = 65 from the Cosine Rule.

This is an inspiration from a Coffin problem

Ong Zi Qian
Jan 26, 2019

Draw triangle A F C A E B AFC \cong AEB ,

E A F = E A C + C A F = E A C + B A E = B A C = 6 0 \angle EAF = \angle EAC + \angle CAF = \angle EAC + \angle BAE =\angle BAC =60^\circ \rightarrow Triangle EAF is equilateral triangle. E F = F A = A E = 5 EF=FA=AE=5 units.

Noted that E F 2 + E C 2 = F C 2 EF^2+EC^2=FC^2 , by converse of Pythagorean theorem, F E C = 9 0 \angle FEC = 90^\circ

Using cosine rules, we have A C 2 = A E 2 + E C 2 2 A E E C cos ( 6 0 + 9 0 ) = 4225 AC^2 = AE^2+EC^2-2AE\cdot EC \cos(60^\circ+90^\circ)=4225

hence A C = 65 AC=65

Rab Gani
Jan 22, 2019

Apply Pythagoras theorem: Let a is horizontal distance form the vertical symmetry line, b is vertical distance from apex to Brilli. ((1/2)x + a)^2 + ((1/2)x√3 – b)^2 = (35√3)^2, ...........(1) ((1/2)x - a)^2 + ((1/2)x√3 – b)^2 = (10√37)^2, ...........(2) a^2 + b^2 = 5^2. ...........(3), Substract eqs(1) by eqs(2), we get 2xa = -25, or a = (-25)/(2x) , Add eqs(1) to eqs(2), we get 2x^2 + 50 - 2√3xb = 7375, or b = - (7325-2x^2)/(2√3x)
Substitute a, and b to eqs(3) and simplify, then we get x^4 -7400 x^2+ 13414375 = 0, then x^2= (7400±1050)/2 , or x=65

0 pending reports

×

Problem Loading...

Note Loading...

Set Loading...